What is the percentage decrease from 80 to 60?; What is the percentage decrease of 500 to 240?; How do you find the percentage of a decrease?; What is the percent of decrease from 144 to 120?

Answers

Answer 1

Using simple percentage rule, the answer is 25%,52% and 16.67%.

What is mean by percentage?

A percentage is a figure or ratio stated as a fraction of 100 in mathematics. The percent symbol, %, or the abbreviation "pct" are frequently used to indicate percentages. The term "percent" derives from "percent," a shortened form of "per centum," which meaning per hundred.

What is the formula to calculate percentage?

The required formula is:

percentage (%) = Change/total * 100

Percentage Decrease = [tex]\frac {80-60}{80} * 100[/tex]

=25%

Percentage Decrease=[tex]\frac {500-240}{500} * 100[/tex]

=52%

Percentage Decrease = [tex]\frac {144-120}{144} * 100[/tex]

=16.67%

We calculated the percentage decrease using the above-mentioned formula.

To learn more about fractions and percentage visit:

https://brainly.com/question/10354322

#SPJ4


Related Questions

6 suppose a die is tossed 1000 times, and the following frequencies are obtained for the number of pips up when the die comes to a rest. x1 x2 x3 x4 x5 x6 163 178 142 150 183 184 using the chi-squared goodness of fit test, assess whether we have evidence that this is not a symmetrical die. record the standardized residuals

Answers

We have evidence that this is not a symmetrical die the standardized residuals is 9.5699.

Given that,

The following frequencies are found for the number of pips up when the die comes to rest, assuming a die is tossed 1000 times. Find out if there is proof that this die is not symmetrical using the chi-squared goodness of fit test. The standardized residuals should be noted.

x₁           x₂           x₃           x₄         x₅           x₆

163        178        142        150       183        184

We know that,

          observed       expected         Op-Ep         (Op-Ep)²/Ep

x₁          163                 166.7                -3.7               0.082

x₂          178                 166.7                 11.3              0.7659

x₃          142                 166.7                -24.7            3.6598

x₄          150                 166.7                -16.7             1.673

x₅          183                 166.7                16.3               1.5938

x₆           184                 166.7               17.3               1.7954

                                                                                 -----------

                                                                                 9.5699

Expected = 1/nΣXa=1/6(1000)=166.7

Therefore, We have evidence that this is not a symmetrical die the standardized residuals is 9.5699.

To learn more about standard visit: https://brainly.com/question/23907081

#SPJ4

find the absolute maximum and absolute minimum of f (x, y) among points in the triangle with vertices (0, 0), (1, 0), and (1, 1).

Answers

The absolute maximum and absolute minimum of f(x,y) = 3 + xy -x - 2y is f(0,0) and f(1,1) respectively.

The absolute maximum point is a point where the function obtains its greatest possible value.

And Absolute minimum is a point where the function obtains its least possible value. This is the smallest value that a mathematical function can have over its entire curve.

Given equation is, f(x,y) = 3 + xy - x - 2y

we will check absolute maximum and absolute minimum of f(x,y) at the points (0,0), (1,0), (1,1).  

f(0,0) = 3 + (0)(0) - 0 - 2(0) = 3

f(1,0) = 3 + (1)(0) - 1 - 2 (0) = 2

f(1,1) = 3 + (1)(1) - 1 - 2(1) = 1

Therefore we get absolute maximum at f(0,0) and absolute minimum at f(1,1).

An absolute minimum also called a global minimum, occurs when a point of the function is lower any other point on the function within the function's domain. A local minimum also called relative minimum occurs when a point is lower than the points surrounding it.

Given question is incomplete. Complete question is:

find the absolute maximum and absolute minimum of f (x, y) = 3 + xy - x -2y  among points in the triangle with vertices (0, 0), (1, 0), and (1, 1).

To know more about maximum here

https://brainly.com/question/13176730

#SPJ4

evaluate the expression: v ⋅ w given the vectors: r = <8, 8, -6>; v = <3, -8, -3>; w = <-4, -2, -6> (2 points)

Answers

The product of the vectors v and w is gotten as; v ⋅ w = 22

What is the product of the vectors?

We are given the vectors;

r = <8, 8, -6>

v = <3, -8, -3>

w = <-4, -2, -6>

We want to evaluate v.w which is a product of both vectors.

The formula will be;

v.w = (v_x * w_x) + (v_y * w_y) + (v_z * w_z)

Plugging in the values and solving gives;

v.w = (3 * -4) + (-8 * -2) + (-3 * -6)

v.w = -12 + 16 + 18

v.w = 22

We can conclude that the vector expressions has a product of 22

Read more about Vector Products at; https://brainly.com/question/9131049

#SPJ1

When practicing statistics in real life, it is not very important to check the necessary assumptions of a statistical procedure in order to effectively carry out and use the results. True False

Answers

The given statement is FALSE.

Given statement;

When practicing statistics in real life, it is not very important to check the necessary assumptions of a statistical procedure to effectively carry out and use the results.

The above-mentioned statement is FALSE.

→ We are aware that checking assumptions is crucial when applying statistics in daily life because failing to do so will prevent us from receiving fair answers to our statistical queries or drawing valid inferences. To use the good process and appropriate statistical distributions while utilizing statistics in daily life, it is crucial to examine assumptions.

To learn more about statistics click here:

brainly.com/question/23091366

#SPJ4

Tanya has a collection of 52 rocks that she wants to put into display cases. Each display case can hold 8 rocks. Tanya divides to find how many cases
she needs.
52÷8=614
How many display cases will Tanya need?

A. 4

B. 6

C. 7

D. 8

E. 10

Answers

The answer would be (B

Let a, b, and c be real numbers where a b c 0. Which of the following functions could represent the graph below?

Answers

Answer:

[tex]f(x)=x^2(x-a)^2(x-b)^4(x-c)[/tex]

Step-by-step explanation:

There is a double root at [tex]x=0[/tex], which corresponds to [tex]x[/tex] raised to an even power.

There are also 2 more double roots and one more single root. This corresponds to two factors raised to an even power and one factoe raised to an odd power.

The only option that satisfies this is Option 1.

The graphical representation of the polynomial function is given by the relation f ( x ) = x² ( x - a )²( x - b )⁴ ( x - c )

What is Equation of Graph of Polynomials?

Graphs behave differently at various x-intercepts. Sometimes the graph will cross over the x-axis at an intercept. Other times the graph will touch the x-axis and bounce off.

Identify the even and odd multiplicities of the polynomial functions' zeros.

Using end behavior, turning points, intercepts, and the Intermediate Value Theorem, plot the graph of a polynomial function.

The graphs cross or are tangent to the x-axis at these x-values for zeros with even multiplicities. The graphs cross or intersect the x-axis at these x-values for zeros with odd multiplicities

Given data ,

Let the polynomial equation be represented as f ( x )

Now , the value of f ( x ) is

f ( x ) = x² ( x - a )²( x - b )⁴ ( x - c )

The factors (x - a), (x - b), and (x - c) indicate that the function has roots at x = a, x = b, and x = c. These roots are of multiplicity 2, 4, and 1 respectively, which means that they are repeated in the factors (x - a)², (x - b)⁴, and (x - c), respectively.

The factor x² indicates that the graph of the function opens upward, since the leading coefficient is positive.

Overall, the graph of this function will have eight "humps" or "bumps" corresponding to its eight roots, and the heights and shapes of these bumps will depend on the values of a, b, and c.

Hence , the graph of the function is plotted

To learn more about equation of graph of polynomials click :

https://brainly.com/question/16957172

#SPJ5

(1 point) For each of the following pairs of groups G1, G2, determine the number of elements in the direct product G, x G2 of the given order (D6 is the dihedral group of order 12). G G2knumber of elements in G x G2 of order k Z16 Z19 4 U(14) U(13)6

Answers

The number of elements in the direct product are 2.

What is direct product of groups?

The direct product, often written as G x H, is an operation used in group theory, a branch of mathematics, to create a new group from two existing ones. This operation, one of several crucial direct product concepts in mathematics, is the group-theoretic equivalent of the Cartesian product of sets.

Let, the number of elements of order 1 in  [tex]Z_1_6[/tex]  and  [tex]Z_1_9[/tex]  is 1.

Number of elements of order 2 in [tex]Z_1_6[/tex]  and [tex]Z_1_9[/tex]  are 1 and 0 respectively.

Number of elements of order 4 in [tex]Z_1_6[/tex]  and  [tex]Z_1_9[/tex] are 2 and 0 respectively.

Consider the table of number of elements of order.

                           Number of elements of order

                              1         2          4

           [tex]Z_1_6[/tex]              1         1           2

          [tex]Z_1_9[/tex]               1         0          0

Let x ∈ [tex]Z_1_6[/tex] x [tex]Z_1_9[/tex]

O(x) = lcm (O(a), O(b))

where x = (a, b)

That is, a ∈ [tex]Z_1_6[/tex]  and  b ∈ [tex]Z_1_9[/tex]  

Here only (4, 1) satisfies it and [tex]Z_1_6[/tex] has 2 elements of order 4 and [tex]Z_1_9[/tex]  has one element of order 1.

So,  [tex]Z_1_6[/tex] x [tex]Z_1_9[/tex] has 2 x 1 = 2 elements of order 4.

Hence, the number of elements in the direct product are 2.

To know more about direct product of groups, click on the link

https://brainly.com/question/17012757

#SPJ4

Complete question:

Complete question is attached below.

TRUE/FALSE. incidence of multiple chronic conditions is connected to frailty, a condition marked by a lack of resilience to physiological changes and an elevated risk of poor health outcomes.

Answers

The given

Incidence of multiple chronic conditions is connected to frailty, a condition marked by a lack of resilience to physiological changes and an elevated risk of poor health outcomes.

It is TRUE.

"Information available from the question:"

In the question:

Incidence of multiple chronic conditions is connected to frailty, a condition marked by a lack of resilience to physiological changes and an elevated risk of poor health outcomes.

It is TRUE / FALSE

Now, According to the question:

Let's know:

What is Frailty?

Frailty is most often defined as an aging-related syndrome of physiological decline, characterized by marked vulnerability to adverse health outcomes.

Incidence of multiple chronic conditions is connected to frailty, a condition marked by a lack of resilience to physiological changes and an elevated risk of poor health outcomes.

Hence, The above statement is TRUE.

Learn more about Frailty at:

https://brainly.com/question/28287310

#SPJ4

A sprinter completed a 100 m run with a time of 12.31 seconds.
Her time,
t
, has been rounded to the nearest hundredth of a second.
Write the error interval for
t
in the form
a

t
<
b
.

Answers

The error interval for the time spent to complete the race is 12.305 ≤ t < 12.315

How to determine the error interval for the time spent to complete the race

From the question, we have the following parameters that can be used in our computation:

Distance = 100 m

Time = 12.31 seconds

From the question, we understand that

Time = rounded to the nearest hundredth of a second.

The highest number that can be rounded to 12.31 to the nearest hundredth is 12.314

While the least is 12.305

When represented as an interval, we have

12.305 ≤ t < 12.314

Add/subtract the error

So, we have

12.305 - 0.001 ≤ t < 12.314 + 0.001

Evaluate

12.305 ≤ t < 12.315

Hence, the error interval is 12.305 ≤ t < 12.315

Read more about error margin at

https://brainly.com/question/25613341

#SPJ1

what was the mistake that was made?

Answers

Answer: they didnt distribute the 7 to the 4.5

Step-by-step explanation:

when distributing something outside of a parenthesis, you have to distribute it to all terms within the parenthesis, not just the variable

Which statements are true about box plots? Check all that apply.
They help describe sets of data.
They include the mean.
They show the data split into four parts.
They show outliers through really short “whiskers.”
Each section of a box plot represents 25% of the data.

Merry Christmas!

Answers

The true statements are they help describe sets of data, they show the data split into four parts and each section of a box plot represents 25% of the data.

What is the box-and-whisker plot?

A box and whisker plot displays a "box" with its left edge at Q₁, right edge at Q₃, "center" at Q₂ (the median), and "whiskers" at the maximum and minimum.

The box plot is also called box-and-whisker plot.

The data is divided into sections by box plots, with each section containing around 25% of the total data.

Box plots are useful as:

They offer a visual overview of the data,

making it simple for researchers to spot skewness,

the dispersion of the data set.

The minimum value, first quartile, median, third quartile, and maximum value are the five values that make up a box plot.

And they don't include mean.

From the above discussion,

we have three statements that are true and those statements are;

they help describe sets of data, they show the data split into four parts and each section of a box plot represents 25% of the data.

Therefore, the correct statements are A, C and E.

To learn more about the box-and-whisker plot;

https://brainly.com/question/2742784

#SPJ1

If the discriminant of an equation is negative, which of the following is true of the equation.

Answers

Equations have two complex solutions if their discriminant is negative. So, option A is the correct answer.

In algebra, the discriminant of any equation is a quantity that is used to find out about the nature of the roots using the formula = b²-4ac. It tells us if there are no solutions, one solution, or two.

Here, we have been told that the discriminant is negative.

From the formula, we can say that = D = b²-4ac < 0.

Here, if D<0, then x becomes an imaginary value. This means that the roots are going to be imaginary i.e., complex. Thus, the two roots that we get as a result of the formula are both going to be complex.

So, option A is the answer.

The complete that you might be looking for is given below.

If the discriminant of an equation is negative, which of the following is true of the equation

A. it has two complex solutions

B. it has one real solution

C. it had two real solutions

Learn more about the Discriminant on

https://brainly.com/question/15884086?referrer=searchResults

#SPJ4

Answer:

a.two complex solutions

Step-by-step explanation:

y <= -15x + 3000y <= 5xIn the xy plane, if a point with coordinates (a,b) lies in the solution set of the system of inequalities above, what is the maximum possible value for b?

Answers

B can have a maximum value of 750.

Given info,

y ≤ - 15x + 3000

y ≤ 5x

What is the highest possible value of b in the XY plane if a point with the coordinates (a,b) is found in the system of inequalities above?

The maximum value of b occurs when taking equality.

⇒ - 15x + 3000 = 5x

⇒ - 20x = - 3000

⇒ - 20x = - 3000

⇒ x = 150

Therefore, y = 5 * 150

                    = 750

Hence, the maximum possible value of b = 750

To learn more about equality click here:

brainly.com/question/9070018

#SPJ4

Amanda is spreading new grass seed on her yard. The area of her yard is 4,995 square feet. The instructions on the bags of grass seed say that one bag of seed should be used for every 100 square yards. How many bags of grass seed should Amanda use?
Group of answer choices

Answers

The number of bags of grass seed that Amanda should use to spread her yard is 16.65 bags, meaning she needs to buy not more than 17 bags, using mathematical operations.

What are mathematical operations?

The basic mathematical operations we use are subtraction, addition, division, and multiplication.

In this situation, we use the division operation.

The division operation involves four parts:

The dividend (numerator or the number being divided).The divisor (denominator or the number dividing).The quotient (the result of the division operation).The equal symbol (that shows that two values are equal).

The area of the yard = 4,995 ft²

1 yard = 3 feet

Area of the yard = 1,665 square yards(4,995/3)

The number of bags per 100 square yards = 1 bag

For 1,665 square, the number of bags = 16.65 bags (1,665 ÷ 100)

Thus, Amanda requires 16.65 bags of the grass seed for her yard.

Learn more about mathematical operations at https://brainly.com/question/20628271

#SPJ1

if x= -1 is a root of x3+3x2-x-3=0 use synthetic division to factor the polynomial completely

Answers

The factors of the polynomial x³+3x²-x-3 are (x+1)(x-1)(x+3).

What are factors?

A factor is a number that divides another number, leaving no remainder.

Given is a polynomial, x³+3x²-x-3

x = -1 is the root of the polynomial, which it will completely divide the polynomial x³+3x²-x-3,

On dividing, we get,

(x³+3x²-x-3)÷(x+1) = (x²+2x-3)

Factorizing, (x²+2x-3), we get,

(x-1)(x+3)

Hence, The factors of the polynomial x³+3x²-x-3 are (x+1)(x-1)(x+3).

For more references on factors, click;

https://brainly.com/question/14067703

#SPJ1

I need help with this please help me

Answers

It’s -1,0, since it’s only moving one placement backwards, and there is no movement up and down

a chain of sport shops catering to beginning skiers, headquartered in aspen, colorado, plans to conduct a study of how much a beginning skier spends on his or her initial purchase of equipment and supplies. based on these figures, it wants to explore the possibility of offering combinations, such as a pair of boots and a pair of skis, to induce customers to buy more. a sample of 44 cash register receipts revealed these initial purchases: $ 180 $ 186 $ 130 $ 263 $ 258 $ 159 $ 127 $ 84 $ 208 180 104 233 92 223 176 216 219 209 205 150 241 110 239 124 173 174 264 232 182 261 216 193 196 223 239 176 261 150 245 84 232 152 95 233 click here for the excel data file required: a. arrive at a suggested class interval. (round your answer up to the next multiple of 5.) b. organize the data into a frequency distribution using a lower limit of $70.

Answers

(a) Class interval of a given data is 36.

(b) Frequency distribution is 44.

We have given that,

total frequency = 44

lower limit = $70

number of class = 5

(a) we have to find class interval,

Class interval =  [tex]\frac{Max. number - min. number}{no. of classes}[/tex]

here maximum number is 264 and minimum number is 84

Class interval = [tex]\frac{264 - 84}{5}[/tex]

                      = [tex]\frac{180}{5}[/tex]

Class interval = 36

we have to round our value to the next multiple of 5

so the class interval will be 40.

(b) A frequency distribution is the pattern of frequencies of a variable. It is the number of times each possible value of a variable occurs in a dataset. There are three types of frequency distribution such as Cumulative frequency distribution, Relative frequency distribution, Relative cumulative frequency distribution.  

The frequency distribution is as follows:

Class interval           Frequency

 70 - 110                         6

 111 - 151                          5

152 - 192                       10

193 - 233                       14

234 -  274                       9

Total                              44

This is the organize data into a frequency distribution.

To know more about class interval here

https://brainly.com/question/28367164

#SPJ4


A 3-gallon container of window cleaner costs $17.76. What is the price per quart?

Answers

Answer: $1.48

Step-by-step explanation:

find how much each gallon cost

1. 17.76/3=5.92

Divide by 4 because 4 quarts equal a gallon

2. 5.92/4=1.48

Therefore each quart equals $1.48,

Hope this helps!

If X is a metric space, and fn:X→R (n∈N) is a sequence of functions, then fn converges pointwise to f if for every x∈X one has limn→[infinity]fn(x)=f(x).

Answers

|f(x) − f n(x)| = lim m→∞

|f m(x) − f n(x)| ≤ ϵ /2< ϵ

Converges Uniformly

A sequence of functions f n(x); n = 1, 2, 3,…. Is said to be uniformly convergent to f for a set E of values of x, if for each ε > 0, a positive integer N exists such that |f n(x) – f(x)| < ε for n ≥ N and x ∈ E.

A sequence (f n) of functions f n : A → R is uniformly Cauchy

on A if for every ϵ > 0 there exists N ∈ N such that

m, n > N implies that |f m(x) − f n(x)| < ϵ for all x ∈ A.

The key part of the following proof is the argument to show that a pointwise

convergent, uniformly Cauchy sequence converges uniformly.

A sequence (f n) of functions f n : A → R converges uniformly on

A if and only if it is uniformly Cauchy on A.

Suppose that ( f n) converges uniformly to f on A. Then, given ϵ > 0, there

exists N ∈ N such that

|f n(x) − f(x)| < ϵ/2

for all x ∈ A if n > N.

It follows that if m, n > N then

|f m(x) − f n(x)| ≤ |f m(x) − f(x)| + |f(x) − f n(x)| < ϵ for all x ∈ A,

which shows that (f n) is uniformly Cauchy.

Conversely, suppose that (f n) is uniformly Cauchy. Then for each x ∈ A, the

real sequence (f n(x)) is Cauchy, so it converges by the completeness of R. We

define f : A → R by

f(x) = limn→∞

f n(x),

and then f n → f pointwise.

To prove that f n → f uniformly, let ϵ > 0. Since (f n) is uniformly Cauchy,

can choose N ∈ N (depending only on ϵ) such that

|f m(x) − f n(x)| < ϵ /2

for all x ∈ A if m, n > N.

Let n > N and x ∈ A. Then for every m > N we have

|f n(x) − f(x)| ≤ |f n(x) − f m(x)| + |f m(x) − f(x)| < ϵ /2 + |f m(x) − f(x)|.

Since f m(x) → f(x) as m → ∞, we can choose m > N (depending on x, but it

doesn’t matter since m doesn’t appear in the final result) such that

|f m(x) − f(x)| < ϵ/2.

It follows that if n > N, then

|f n(x) − f(x)| < ϵ for all x ∈ A,

Then proves that f n → f uniformly

Alternatively, we can take the limit as m → ∞ in the Cauchy condition to get

for all x ∈ A and n > N that

|f(x) − f n(x)| = lim m→∞

|f m(x) − f n(x)| ≤ ϵ /2< ϵ

To learn more about Converges Uniformly visit:

brainly.com/question/29736068

#SPJ4

In how many ways can Tim draw a red or a green marble from a jar containing 20 red marbles and 4 green marbles?

Answers

The correct answer to how many different ways he can draw is 24.

Write an equation of the line that passes through (-1, 8) and is parallel to the line y = 6.

Answers

The equation of the line parallel to y = 6 and passing through (-1, 8) is y = 8.

What is slope?

The slope or gradient of a line is a number that describes both the direction and the steepness of the line.

Given that, an equation of the line that passes through (-1, 8) and is parallel to the line y = 6.

Slope of the parallel lines are equal to each other, therefore, solving for c,

8 = -1x0 + c

c = 8

Hence, The equation of the line parallel to y = 6 and passing through (-1, 8) is y = 8.

For more references on slope, click;

https://brainly.com/question/3605446

#SPJ1

Let A be set of all prime numbers, B be the set of all even prime numbers, C be the set of all odd prime numbers, then which of the following is true? a) A = BUC b) B is a singleton set. c) A = CU{2} d) All of the mentioned

Answers

After analyzing the given data we will get to know that all the options mentioned above are true.

A = BUC, B is a singleton set, and A = CU2 if A is the set of all prime numbers, B is the set of all even prime numbers, and C is the set of all odd prime numbers. Therefore, option D - All of the specified - is the answer to this question.

We've got

The entire set of prime numbers is A.

A = {2, 3, 5, 7, 11, 13}

B is a collection of even prime numbers.

B = {2}

C is a collection of all odd prime numbers.

C = {3, 5, 7, 11, 13,}

We are aware that the only even prime number is two. As a result, Set B is a singleton set because it only has one element.

Additionally, if we combine sets B and C, we will obtain set A, which contains all prime numbers.

Thus, we obtain A = B U C.

Since B = 2 is a singleton set, if we have A = B U C, we can also say A = C U 2.

Therefore, each of the aforementioned choices is appropriate.

To learn more about the prime numbers

brainly.com/question/25110414

#SPJ4

How much would Howard Steele need to invest today so that he may withdraw $12,000 each year for the next 20 years, assuming a rate of 8% compounded annually?
Multiple Choice
$117,817.77
$454,144.00
$112,817.20
$549,144

Answers

Howard Steele needs to invest  $117,817.77 today so that he may withdraw $12,000 each year for the next 20 years

What are simple and compound interests?

Simple interest is often a predetermined percentage of the principal amount borrowed or lent paid or received over a specific time period.

Borrowers are required to pay interest on interest in addition to principal since compound interest accrues and is added to the accrued interest from prior periods.

We know the equation, [tex]P = \frac{A\times(1-(1+r)^{-n}}{r}[/tex] the annuity, and the present value of money, P, can be connected.

P = 12000, r = 0.08 and n = 20.

∴ [tex]12000 = \frac{A\times(1-(1.08)^{-20}}{0.08}[/tex].

A = $117,817.77.

learn more about compound interest here :

https://brainly.com/question/29335425

#SPJ1

Determine if x and y are inversely proportional. If applicable, find the constant of variation.

Answers

The relationship between x and y is and the constant of variation is 24.

What is inversely proportional?

One kind of proportionality relationship is inverse proportion. If two quantities are inversely related, one will increase while the other will decrease. Building a wall would be an example of an inverse proportion in action.

Given that x and y are inversely proportion.

Assume that the relationship between x and y is

y = k/x where k is constant of variation.

Putting x = 3 and y = 8 in y = k/x :

8 = k/3

Multiply both sides by 3:

k = 8 × 3

k = 24

Putting x = 4 and y = 6 in y = k/x :

6 = k/4

Multiply both sides by 4:

k = 6 × 4

k = 24

The relationship is y = 24/x and the constant of variation is 24.

To learn more about constant of variation, click on below link:

https://brainly.com/question/20355158

#SPJ1

Answer:

x and y are not inversely proportional.

Image:

A pharmacy has determined that a healthy person should receive 70 units of
proteins, 100 units of carbohydrates and 20 units of fat daily. If the store carries
the six types of health food with their ingredients as shown in the table below,
what blend of foods satisfies the requirements at minimum cost to the
pharmacy? Make a mathematical model for the given problem.
Foods Protein units Carbohydrates
units
Fat units Cost per unit
A 20 50 4 2
B 30 30 9 3
C 40 20 11 5
D 40 25 10 6
E 45 50 9 8
F 30 20 10 8

Answers

optimal solution is arrived with value of variables as :

x1=0.9091,x2=1.8182,x3=0,x4=0,x5=0,x6=0

minimize cost:

z=2x₁+3x₂+5x₃+6x₄+8x₅+8x₆=2x

subject to:

20x₁+30x₂+40x₃+40x₄+45x₅+30x₆ ≥70 : amount of protein

50x₁+30x₂+20x₃+25x₄+50x₅+20x₆ ≥100 : amount of carbohydrate

4x₁+9x₂+11x₃+10x₄+9x₅+10x₆ ≥20 : amount of fat

where x₁, x₂, x₃, x₄, x₅, x₆ are units of 6 foods

solution using Simplex method:

After introducing artificial variables:

Min Z=2x₁+3x₂+5x₃+6x₄+8x₅+8x₆+0S₁+0S₂+0S₃+MA₁+MA₂+MA₃

subject to

20x₁+30x₂+40x₃+40x₄+45x₅+30x₆-S₁+A₁ ≥ 70

50x₁+30x₂+20x₃+25x₄+50x₅+20x₆-S₂+A₂≥ 100

4x₁+9x₂+11x₃+10x₄+9x₅+10x₆-S₃+A₃=20

and x₁,x₂,x₃,x₄,x₅,x₆,A₁,A₂,A₃≥0

Refer to the image for Z

Since all Zj-Cj≤0

Hence, optimal solution is arrived with value of variables as :

x1=0.9091,x2=1.8182,x3=0,x4=0,x5=0,x6=0

Min Z=7.2727

Learn more about Simplex Method at:

https://brainly.com/question/14410494

#SPJ1

the point (-4,-6,10) lies on the surface of a sphere with centre (2,3,-1). hence find the volume of the sphere

Answers

The radius of the sphere with the given point and the center is √238.

Here we have to find the radius of the sphere.

Given data:

The point on the surface of a sphere is (-4, -6, 10) and the point on the center is (2, 3, -1).

The formula for the radius:

r² = (x2 - x1)² + (y2 - y1)² + (z2 - z1)²

where (x1, y1, z1) = point on the center

(x2, y2, z2) = point on the sphere

Now putting the values we have

r² = (-4-2)² + ( -6-3)² + ( 10 +1)²

  =( -6)² +(- 9)²  + (11)²

  = 36 + 81 + 121

  = 238

r = √238.

Therefore the radius is √ 238.

To know more about the sphere refer to the link given below:

https://brainly.com/question/22807400

#SPJ4

Help Please...
Most adult medication doses are for a person weighing 150 pounds. For a 45-pound child, the adult dose should be multiplied by 0.3. If a child's dose of a decongestant is 9 milligrams, what is the adult dose? Please show your work.

Answers

The adult dose for the decongestant is 2.7 milligrams.

How to calculate the adult dose?

The information can be illustrated through an expression. Expression simply refers to the mathematical statements which have at least two terms which are related by an operator and contain either numbers, variables, or both. Addition, subtraction, multiplication, and division are all possible mathematical operations.

Since for a 45-pound child, the adult dose should be multiplied by 0.3, When the child's dose of a decongestant is 9 milligrams, the adult dose will be:

= Children dose × 0.3

= 9 × 0.3

= 2.7

Learn more about expressions on;

brainly.com/question/723406

#SPJ1

Find the area of the triangle. [?] units² Round to the nearest hundredth.​

Answers

The area of the triangle is 48 square units.

To calculate the length of each side of the triangle ACB, the radii of the circles should be added respectively.

Length of side BC = 9+3 = 12 units

Length of side CA = 3+5 = 8 units

Length of side BA = 9+5 = 14 units

Area of triangle = (1/2 × Base × Height) square units

Area of triangle ACB = 1/2 × CA× BC sq units

                          Area = 1/2 × 8 × 12 = 48 sq units

Hence, the area of triangle ACB is 48 square units.

Learn more about the area of triangles here:

https://brainly.com/question/28884904

Answer:

47.91 units²

Step-by-step explanation:

The radius of each circle is:

Circle A:  radius = 5Circle B:  radius = 9Circle C:  radius = 3

Therefore, the side lengths of the triangle are:

AB = 5 + 9 = 14BC = 9 + 3 = 12AC = 3 + 5 = 8

Heron's Formula allows us to find the area of a triangle in terms of its side lengths.

[tex]\boxed{\begin{minipage}{8 cm}\underline{Heron's Formula}\\\\$A=\sqrt{s(s-a)(s-b)(s-c)}$\\\\where:\\ \phantom{ww}$\bullet$ $A$ is the area of the triangle. \\ \phantom{ww}$\bullet$ $a, b$ and $c$ are the side lengths of the triangle. \\ \phantom{ww}$\bullet$ $s$ is half the perimeter.\\\end{minipage}}[/tex]

The perimeter of a two-dimensional shape is the distance all the way around the outside. Therefore, half the perimeter of the triangle is:

[tex]\implies s=\dfrac{14+12+8}{2}=17[/tex]

To calculate the area of the triangle, substitute the sides lengths and the value of s into Heron's formula and solve for A:

[tex]\implies A=\sqrt{17(17-14)(17-12)(17-8)}[/tex]

[tex]\implies A=\sqrt{17(3)(5)(9)}[/tex]

[tex]\implies A=\sqrt{2295}[/tex]

[tex]\implies A=47.906158268...[/tex]

[tex]\implies A=47.91\; \sf units^2\;\;(nearest\;hundredth)[/tex]

Given the equation, proof it

Answers

Answer:

Step-by-step explanation:

I forgot to add the last reason is AAS (two angles one side are congruent = triangles are congruent)

S.

1.Given

2. AE = EC

3. <AEB = <CED

4. <CDE = <ABE

5. triangles congruent

R.

1. Given

2. Segment bisector theorem

3. Vertical angles theorem

4. PAI Theorem

Hope this helps!

One of the 55 students are selected at random.written as a fraction what is the probability

Answers

If one of the 55 students is selected at random, the probability of this occurring can be written as a fraction by dividing the number of selected students by the total number of students. In this case, the probability would be 1/55.

Other Questions
which cytokines are believed to contribute to autoimmune disease, such as rheumatoid arthritis? which cytokines are believed to contribute to autoimmune disease, such as rheumatoid arthritis? chemokines interferons tumor necrosis factor (tnf) hematopoietic cytokines natalie encourages her employees by recognizing and immediately rewarding a good performance, so that it motivates the employee as well as others to consistently perform well. this type of strategy to increase the frequency of desirable behavior is known as . The element carbon is contained in all organic compounds.Block:a. Discuss the role of photosynthesis and cellular respiration in carbon cycling in the biosphere. In other words, how are cellular respiration and photosynthesis interdependent? although a strong romantic relationship benefits a couples happiness, research shows that loneliness can be experienced if time spent with friends is curtailed or eliminated. A 0.5 kg ball has a velocity of 20 m/s.a. What is the kinetic energy of the ball?B. How much work would be required to stop the ball which of the following expresses the relationship for the change in entropy in a reversible process between two end states, and then the change in entropy in an irreversible process between the same two end states? A) (S2-S1) irreversible = (S2-S1) reversible B) (S2-S1) irreversible > (S2-S1) reversible C) (S2-S1) irreversible < (S2-S1) reversible D) None of the above the secretion of aldosterone is brought about by all these factors except one. which factor is incorrect? t cells are primarily resopnsible for at a baseball game the batter hits a ground ball for a base hit and runs to first base. true or false: the batter's distance from the first base bag is a function of time. true or false Identify the statistical test in which the mean difference between two groups are compared to a distribution of differences between means.a.independent-samples t testb.paired-samples t testc.z testd.single-sample t test Unlike perfectly markets, health insurance and health care markets contain asymmetric information in many forms. To see the consequences, consider the following model The population is evenly divided between 2 types of people: healthy people and unhealthy people. Healthy people have expected health care costs at S1000per year. Unhealthy people have expected health care costs of $5000 per year. Unhealthy people can become healthy by working out, eating healthier, and taking entive care. Assume that the cost becoming healthy, in terms of time and effort is $2000 per year hese people live in a city with one employer who will hire anyone who is willing to work. This employer provides complete health care to all its employees, all health care costs are covered by the insurance, What is the actuarial fair oost of Do the employees have incentive insurance for all the workers? to become hea O Yes O Not enough information. O No What would the new actuarially fair cost of a new employer enters who pays S1500 If insurance be at the original firm? surance), which people will go and work for the new employer? O None O Unhealthy O Both O Healthy O Previous ove up s view 2Check Answer terms of use contact us br warm-up type your response in the box. analyze the following scenarios and think about what is causing these economies to improve. scenario 1 country a invested in reliable wireless internet for most of its cities. as a result, more people and small businesses have access to high-speed internet. households are happy with this development, as they now have improved wi-fi for communicating, downloading movies, gaming, and so on. firms are even happier because their bill payment, sales, and customer service processes are much faster and more reliable. output has even increased in local manufacturing firms because automated machines run quicker. scenario 2 country b invested in free, mandatory public education for boys and girls ages 6 through 18. children will also have access to free transportation to and from school. while some people criticize the government for this tremendous expense, the government believes it will recover the investment when the country has a more educated, well-trained workforce. what is driving economic growth in each of these countries? describe one economic impact that the mit'a system had on europe Can oxygen and sulfur form an ionic bond? does rna polymerase move in a set direction along a strand of dna during transcription? a yes, the rna polymerase moves in a direction that reads the bases of the dna sequence from the 5' end toward the 3' end. b yes, the rna polymerase moves in a direction that reads the bases of the dna sequence from the 3' end toward the 5' end. c no; the rna polymerase can move in either direction along the dna strand because the same sequence of bases in the rna will be produced regardless of direction. Find the equation for a polynomial f(x) that satisfies the following:Degree 3Zero at x=1Zero at x=4Zero at x=1y-intercept of (0,8)f(x)= a circle has a radius of 9 in . find the length of the arc intercepted by a central angle of 1.1 radians. do not round any intermediate computations, and round your answer to the nearest tenth. Which one of the following will exhibit the highest osmotic pressure at 25C?a) KCl.b) Glucose.c) Urea.d) Calcium chloride. what is recommended prime number p in (in bits) for diffie - hellman key exchange for security purposes? which of the following is most likely if employees are from cultures with low uncertainty avoidance?